Retirement Planning

Pataasin ang iyong marka sa homework at exams ngayon gamit ang Quizwiz!

If a participant's accrued benefit from a qualified defined benefit pension plan is $2,000 per month, what is the maximum life insurance death benefit coverage that the plan can provide based on the 100 to 1 ratio test? a. $0. b. $2,400. c. $200,000. d. $240,000.

c. $200,000.

Which of the following statements is true? a. A cash balance pension plan usually benefits older employees the most. b. A defined benefit plan promises a contribution to a hypothetical account each year for a plan participant. c. Cash balance pension plan participants under age 62 may take a withdrawal from the plan during employment with the plan sponsor. d. A cash balance pension plan does not have individual separate accounts for each participant.

d. A cash balance pension plan does not have individual separate accounts for each participant.

Which one of the following statements is true for a defined benefit plan? a. A defined benefit plan generally favors older age entrants. b. The maximum retirement benefit payable from a defined benefit plan is the lesser of 100 percent of the participant's compensation or $210,000 for 2014. c. A defined benefit plan with 100 employees is required to pay PBGC insurance premiums. d. All of the above are true.

d. All of the above are true.

Which of the following is not a common defined benefit plan funding formula? a. Flat amount formula. b. Flat percentage formula. c. Unit credit formula. d. Excludible amount formula.

d. Excludible amount formula.

Which of the following statements is true regarding the anti-cutback rule, as it is referred to? a. The anti-cutback rule prohibits the plan sponsor from making changes to the plan that are retroactive to the beginning of the year. b. The anti-cutback rule prohibits the plan sponsor from freezing the benefits under the plan. c. The anti-cutback rule prohibits the plan sponsor from terminating the plan. d. The anti-cutback rule prohibits the plan sponsor from amending the plan such that prior accrued benefits are reduced.

d. The anti-cutback rule prohibits the plan sponsor from amending the plan such that prior accrued benefits are reduced.

Accent, Inc. sponsors a 25% money purchase pension plan for its eligible employees. Carlos earns $200,000, Kevin earns $60,000, Kelly earns $300,000, and Rick, who is ineligible, earns $27,000. What is Accent's required deductible contribution for the year? All employees are under age 50. a. $117,000. b. $126,250. c. $127,500. d. $140,000.

a. $117,000.

Rick, who is age 45, is having a few leadership issues and must resign from his current position as CEO. He worked for A-Send, which sponsors a cash balance plan and a standard 401(k) plan. Each of the plans uses the longest permitted vesting schedule and neither plan is top heavy. He has a balance of $80,000 in the cash balance plan, has deferred $40,000 into the 401(k) plan and has employer matching contributions of $20,000. If he has been employed for two and a half years, but only participating in the plans for the last two years, how much does he keep if he resigns and terminates his employment today? a. $44,000. b. $60,000. c. $124,000. d. $140,000.

a. $44,000.

A defined benefit pension plan has a funding formula equal to 1% x years of service x final salary. If Jim's final salary is $600,000 and Jim has earned 30 years of service, what is Jim's retirement benefit in 2014? a. $78,000. b. $180,000. c. $210,000. d. $260,000.

a. $78,000.

Which of the following statements regarding the plan sponsor of a money purchase pension plan is correct? a. The plan sponsor is required to make an annual contribution to the plan. b. The excess earnings of a money purchase pension plan are returned to the plan sponsor. c. The plan sponsor generally bears the investment risk of the plan assets. d. A plan sponsor with fluctuating cash flows would adopt a money purchase pension plan.

a. The plan sponsor is required to make an annual contribution to the plan.

Knowledge Star is a 30-year-old company that has grown significantly in terms of revenue and product offerings. They sponsor a pension plan that provides a benefit of 2% times years of participation times the average of the final three years of salary less an offset. The offset equals 1% times years of service times income below the covered compensation limit of $40,000 (assumed). Roberto has worked with Knowledge Star for the last 30 years and earned $90,000 two years ago, $110,000 last year, and $130,000 this year. If he is retiring this year, how much should he expect to receive as a pension benefit? a. $45,000. b. $54,000. c. $66,000. d. $78,000.

b. $54,000.

Ashley is a participant in her law firm's defined benefit plan. Ashley, who is an aggressive attorney, is 44 years old and earns $150,000. She has four years of service for purposes of the plan and has worked at the firm for four years. The plan provides a benefit of 1% for the first three years of service and a benefit of 1.5% for all additional years of service. The plan has the least generous vesting schedule possible. Almost eighty percent of the accrued benefits are attributable to the two equal owners, Martin and Bob, who have been working at the company for decades. If Ashley were to leave, what percent of her salary (as defined by the plan) could she expect to receive at normal retirement? a. 3.6%. b. 4.8%. c. 5.5%. d. 6.4%.

b. 4.8%.

Which of the following statements regarding defined benefit plans is true? a. A defined benefit plan can allocate forfeitures to other plan participants. b. A defined benefit plan can use forfeitures to reduce future plan costs. c. A defined benefit plan cannot give credit for prior service. d. Each participant of a defined benefit plan has an individual account.

b. A defined benefit plan can use forfeitures to reduce future plan costs.

Which of the following is not a characteristic of pension plans? a. Mandatory funding. b. In-service withdrawals for employees under the age of 62. c. Limited investment in life insurance. d. A limit of 10 percent investment in the employer's securities.

b. In-service withdrawals for employees under the age of 62.

Of the following statements regarding target benefit pension plans, which is true? a. A target benefit pension plan is a money purchase pension plan with a funding formula that considers age and salary. b. The plan sponsor of a target benefit pension plan does not guarantee that the participant will receive an amount, expected to be the "target benefit" amount, at his retirement. c. Plan participants of a target benefit pension plan do not have separate accounts. d. The plan sponsor guarantees an earnings rate for the contributions made to target benefit pension plans.

b. The plan sponsor of a target benefit pension plan does not guarantee that the participant will receive an amount, expected to be the "target benefit" amount, at his retirement.

Which of the following statements regarding target benefit pension plans is true? a. A target benefit pension plan cannot allocate plan forfeitures to remaining plan participant accounts. b. Target benefit pension plans may not be established after 2001. c. Assuming equal salaries, a target benefit pension plan would allocate a higher percentage of its current contributions to an older employee. d. A target benefit pension plan may always exclude any participant who has not attained the age of 26 and completed one year of service.

c. Assuming equal salaries, a target benefit pension plan would allocate a higher percentage of its current contributions to an older employee.

Which of the following actuarial assumptions is not used by the actuary who determines the mandatory funding range for a defined benefit plan? a. Mortality. b. Turnover. c. Divorce rate. d. Disability rate.

c. Divorce rate.

Bobby is the owner of Ideal Mechanics, Inc. He would like to establish a qualified pension plan and would like most of the plan's current contributions to be allocated to his account. He does not want to permit loans and he does not want Ideal to bear the investment risk of the plan's assets. Bobby is 47 and earns $300,000 per year. His employees are 25, 29, and 32 and they each earn $25,000 per year. Which of the following qualified pension plans would you recommend that Bobby establish? a. Defined benefit pension plan. b. Cash balance pension plan. c. Money purchase pension plan. d. Defined benefit pension plan using permitted disparity.

c. Money purchase pension plan.

Which of the following pension plans would allocate a higher percentage of the plans' current costs to a certain class or group of eligible employees? 1. Defined benefit pension plan. 2. Target benefit pension plan. 3. Money purchase pension plan with permitted disparity. a. 1 only. b. 1 and 2. c. 2 and 3. d. 1, 2, and 3

d. 1, 2, and 3


Kaugnay na mga set ng pag-aaral

PrepU Patho Ch. 18 Disorders of Blood Flow and Blood Pressure

View Set

MKT Research & Analysis Chaudhry Test 2

View Set

HL English Midterm: Streetcar Scene Questions

View Set

Psych: Chapter 9 Legal Issues, Psych CH9

View Set

Health Assessment Final Review (Prep U)

View Set

Science 600 - Unit 5: Chemical Structure and Change Quiz 2: Periodic Table

View Set